Answer to Question #93024 in Electric Circuits for norah

Question #93024
Three cells (EMF =9V) are connected in series to a 4 ohms light bulb. If the resulting current is 0.65 A, what is the internal resistance of each cell, assuming they are identical and neglecting the wires?
1
Expert's answer
2019-08-22T09:23:30-0400

"r=\\frac{\\frac{E}{I}-R}{3}=\\frac{\\frac{9V}{0.65A}-4\\Omega}{3}\\approxeq3.28 \\Omega"

Answer: 3.28 ohms


Need a fast expert's response?

Submit order

and get a quick answer at the best price

for any assignment or question with DETAILED EXPLANATIONS!

Comments

No comments. Be the first!

Leave a comment

LATEST TUTORIALS
New on Blog
APPROVED BY CLIENTS